Talk:2021 Fall AMC 10A Problems/Problem 19

An asymptote diagram for my solution would be appreciated. You can take credit for the diagram!

Note: I cannot do asymptote diagrams. from MathFun1000

Edit: Thanks MRENTHUSIASM!

Yay! :D ~MSAISUHTNERM Victorious-Hedgehog.png